Energy principle example problem in classical mechanics book

AI Thread Summary
The discussion revolves around a physics problem where a man pulls a block using a force that is two-fifths of his weight. The man weighs 100 kg, leading to a maximum pulling force of 400 N, not the 200 N stated in the solution provided. Participants highlight the potential for typos in textbooks and suggest deriving the solution symbolically for clarity. The importance of confirming the work-energy principle through calculations is emphasized. The conversation underscores the need for accuracy in problem statements and solutions in educational materials.
certainice
Messages
1
Reaction score
0

Homework Statement


A man of mass 100 kg can pull on a rope with a maximum force equal to two fifths of his own weight. [Take g = 10 ms^2] In a competition, he must pull a block of mass 1600 kg across a smooth horizontal floor, the block being initially at rest. He is able to apply his maximum force horizontally for 12 seconds before falling exhausted. Find the total work done by the man and confirm that the energy principle is true in this case.

Homework Equations


F*v*dt=W , Work-Energy principle

The Attempt at a Solution



0http://physics.stackexchange.com/questions/279513/energy-principle-example-problem-in-mechanics-book# The problem as stated in the book Classical Mechanics by R Douglas Gregoryhttps://books.google.co.uk/books?id...equal to two fifths of his own weight"&f=false .I only have an issue with a portion of the problem.Part of the solution to that example problem states that the force he applies is a constant 200 N. But shouldn't it be 400 N since the problem states that he pulls with a maximum force of two-fifths of his own weight and his weight is 1000 N and two-fifths *1000 is 400 N? Maybe I am missing something here.
 
Last edited by a moderator:
Physics news on Phys.org
Hi certainice, Welcome to Physics Forums!

It's not unheard of for there to be typos in textbooks, particularly if example problems are "updated" for new editions. 2/5 of 1000 is indeed 400, not 200.

Do the derivation yourself symbolically rather than plugging in numbers, or just show that your own numerical results demonstrate the principle being examined.
 
Thread 'Collision of a bullet on a rod-string system: query'
In this question, I have a question. I am NOT trying to solve it, but it is just a conceptual question. Consider the point on the rod, which connects the string and the rod. My question: just before and after the collision, is ANGULAR momentum CONSERVED about this point? Lets call the point which connects the string and rod as P. Why am I asking this? : it is clear from the scenario that the point of concern, which connects the string and the rod, moves in a circular path due to the string...
Thread 'A cylinder connected to a hanged mass'
Let's declare that for the cylinder, mass = M = 10 kg Radius = R = 4 m For the wall and the floor, Friction coeff = ##\mu## = 0.5 For the hanging mass, mass = m = 11 kg First, we divide the force according to their respective plane (x and y thing, correct me if I'm wrong) and according to which, cylinder or the hanging mass, they're working on. Force on the hanging mass $$mg - T = ma$$ Force(Cylinder) on y $$N_f + f_w - Mg = 0$$ Force(Cylinder) on x $$T + f_f - N_w = Ma$$ There's also...
Back
Top